Bone/Joint and Rheumatology Flashcards

1
Q
  1. J.T. is a 68-year-old woman returning to her primary care practitioner’s office to review the results
    of her most recent dual-energy x-ray absorptiometry
    (DEXA) scan. Her physician reports that her lumbar spine T-score is -2.1 standard deviations (SDs)
    (Z-score -1.1). The physician also reports that J.T. has
    a World Health Organization (WHO) Fracture Risk
    Assessment Tool (FRAX) score of 12% for major
    osteoporotic fracture and 4% for hip fracture. Which
    is best for J.T.’s physician to consider to preserve her
    bone density?
    Bone/Joint and Rheumatology
    ACCP/ASHP 2023 Ambulatory Care Pharmacy Preparatory Review and Recertification Course
    297
    A. Initiate high-dose vitamin D (50,000 international
    units) weekly for 8 weeks and then 2000 units
    daily thereafter.
    B. Initiate calcium carbonate plus vitamin D (600
    mg elemental plus 400 international units) twice
    daily.
    C. Initiate alendronate 35 mg weekly plus calcium/
    vitamin D supplementation.
    D. Initiate alendronate 70 mg weekly plus calcium/
    vitamin D supplementation
A
  1. Answer: D
    According to her DEXA scan results, the patient would
    traditionally be classified as having osteopenia in her
    lumbar spine. In many cases, this would require her to be
    treated only with calcium and vitamin D supplementation. However, because her 10-year risk of a hip fracture is
    greater than 3% with the FRAX tool, the NOF would consider this patient to have osteoporosis and recommend that
    she receive antiresorptive therapy. Of the choices, alendronate is the only agent to have antiresorptive properties,
    and of the two doses, 70 mg once weekly is recommended
    (Answer D is correct; Answers A–C are incorrect).
    Alendronate 35 mg once weekly is considered a prevention
    dose for bisphosphonates.
How well did you know this?
1
Not at all
2
3
4
5
Perfectly
2
Q
  1. D.M. is a 72-year-old woman presenting to her primary care provider for a routine follow-up. At the
    visit, her provider discusses with her that she has been
    taking alendronate 70 mg once weekly for the past 5
    years for osteoporosis (L2/L3 compression fracture
    post-fall). The physician would like to discontinue
    the medication and choose a different medication to
    maintain her bone mineral density (BMD). At her last
    dual-energy x-ray absorptiometry (DEXA) scan, the
    patient’s T-scores were -2.6 and -1.8 at the lumbar
    spine and hip, respectively. These values are relatively unchanged from her baseline. She has no renal
    or hepatic complications, and her metabolic profile is
    within normal limits. Which is best for the patient to
    replace alendronate?
    A. Risedronate 150 mg by mouth once per month.
    B. Raloxifene 60 mg by mouth once per day.
    C. Teriparatide 20 mcg subcutaneous injection once
    per day.
    D. Denosumab 60 mg subcutaneous injection once
    every 6 months
A
  1. Answer: D
    The patient should restart therapy but change to a nonbisphosphonate antiresorptive agent. Using an agent
    for osteoporosis for this patient is important because of
    her history of osteoporosis with fracture. Continuing a
    bisphosphonate is an option, but the likelihood of a serious
    adverse event (MRONJ [medication-related osteonecrosis
    of the jaw], atypical femur fracture) increases with duration
    of bisphosphonate use. Reducing the dose would not be
    appropriate for secondary fracture prevention. Although
    raloxifene is efficacious for secondary fracture prevention, its usefulness is more for preventing breast cancer in
    women at high risk, and it is not cost-effective to use routinely for fracture prevention. Denosumab is the best option
    for this patient because it will maintain the same efficacy
    as a bisphosphonate for fracture prevention (Answer D is
    correct; Answers A–C are incorrect). Unfortunately, we do
    not yet know whether the risk of serious adverse events
    is increased with the duration of sequenced medication
    use (i.e., bisphosphonate to RANKL inhibitor). An anabolic agent should be reserved for future use, more severe
    disease, lack of response, and/or contraindication to antiresorptive therapies.
How well did you know this?
1
Not at all
2
3
4
5
Perfectly
3
Q
  1. C.A. is a 69-year-old woman with rheumatoid arthritis (RA). She is treated with oral methotrexate 15
    mg once weekly, prednisone 10 mg once daily, and
    naproxen 500 mg twice daily as needed. On returning for a follow-up with her rheumatologist, she is
    instructed to decrease prednisone to 7.5 mg once daily
    for another 6 months. A recent DEXA scan reveals an
    11% decrease in her lumbar spine since her DEXA
    about 1 year ago. According to the American College
    of Rheumatology (ACR), which approach is best to
    prevent osteoporosis?
    A. No intervention is required because the patient is
    premenopausal.
    B. Administer calcium carbonate 500 mg plus cholecalciferol 400 units twice daily.
    C. Administer risedronate 150 mg monthly plus calcium and cholecalciferol supplementation.
    D. Administer raloxifene 60 mg once daily plus calcium and cholecalciferol supplementation.
A
  1. Answer: C
    According to the latest edition of the ACR’s guidelines for
    managing glucocorticoid-induced osteoporosis, bisphosphonates should be used for patients older than 40 with
    moderate fracture risk (according to Z-score) if they are
    using 7.5 mg or more of prednisone daily for more than
    6 months. Because the patient meets these criteria, risedronate 150 mg monthly plus calcium and vitamin D
    supplementation is warranted (Answer C is correct;
    Answers A, B and D are incorrect)
How well did you know this?
1
Not at all
2
3
4
5
Perfectly
4
Q
  1. F.R. is a 62-year-old woman with RA. She currently
    uses etanercept 50 mg subcutaneously once weekly
    and ibuprofen 600 mg every 6 hours as needed for
    pain. At her latest visit to her primary care physician’s
    office, she states that she will be traveling abroad later
    this year and needs typhoid vaccination. Which is
    most appropriate for her at this time?
    A. Hold etanercept for 1 month; then vaccinate with
    intramuscular Typhim Vi.
    B. Start vaccination today with oral typhoid vaccine
    (Vivotif).
    C. Vaccinate today with intramuscular typhoid vaccine Typhim Vi.
    D. She is not a candidate for typhoid vaccination.
A
  1. Answer: C
    Patients with RA receiving bDMARDs should not be
    administered live vaccines such as oral typhoid. The intramuscular typhoid vaccination is an inactivated vaccine
    and is therefore safe to administer in patients receiving
    bDMARDs. It is unnecessary to hold biologics before vaccination with an inactivated vaccine (Answer C is correct;
    Answers A, B, and D are incorrect)
How well did you know this?
1
Not at all
2
3
4
5
Perfectly
5
Q
  1. A.T. is a 26-year-old woman who presents to a rheumatologist after being given a diagnosis of RA. She
    has symptoms in her elbows, knees, and hips and
    easily becomes fatigued throughout the day. She has
    difficulty dressing in the morning and has missed 10
    days of work in the past 90 days because of her symptoms. Her laboratory results suggest RA, and she has
    no evidence of blood, liver, or kidney disease. She
    is married but does not plan to have children in the
    foreseeable future. Her medical history is significant
    for menorrhagia, for which she has been using a lowdose ethinyl estradiol/norgestimate monophasic pill
    for the past 3 years. Which medication would be best
    for the patient, according to the 2015 ACR treatment
    recommendations?
    A. Methotrexate 10 mg by mouth once weekly plus
    folic acid 1 mg once daily.
    B. Leflunomide 10 mg by mouth daily.
    C. Adalimumab 40 mg subcutaneously every other
    week.
    D. Tofacitinib 5 mg by mouth twice daily
A
  1. Answer: A
    For this patient, the ACR 2015 treatment recommendations
    encourage providers to use methotrexate as a first-line
    agent for patients presenting within the first 3 months of
    diagnosis. Although leflunomide may be an option, it is
    recommended as an add-on to methotrexate if monotherapy insufficiently controls the patient’s symptoms. A TNF
    inhibitor such as adalimumab could also be an option and
    would have been an option for this patient using the 2012
    guideline update, but it has been moved to second line,
    after failure of methotrexate, for the 2015 iteration (Answer
    A is correct; Answers B-D are incorrect). The JAK inhibitor tofacitinib is recommended for disease-naive patients
    (less than 6 months) whose second-line measures fail or
    for disease-experienced patients (more than 6 months) in
    whom at least a TNF inhibitor fails
How well did you know this?
1
Not at all
2
3
4
5
Perfectly
6
Q
  1. J.P. is a 34-year-old man with a medical history significant for psoriasis. For the past 15 years, he has
    been treated successfully with hydrocortisone cream
    and moisturizers, rarely requiring oral systemic corticosteroids. Today, he presents to his primary care
    physician’s office with a worsening joint pain in his
    hands and elbows. He says the pain is minimal (2/10),
    but annoying. He has been receiving sufficient pain
    relief from naproxen 500 mg twice daily as needed
    but wonders if he could be doing more. On physical
    examination, he has actively inflamed joints in his left
    hand. His physician performs some radiographic evaluations, which reveal signs of axial disease, and the
    physician determines that J.P.’s symptoms are likely
    caused by psoriatic arthritis (PsA). Given the patient’s
    presentation, which is the best regimen for treating his
    arthritic symptoms?
    A. Continue naproxen 500 mg twice daily as needed.
    B. Initiate sulfasalazine 500 mg twice daily.
    C. Initiate etanercept 50 mcg twice weekly.
    D. Initiate etanercept 50 mcg twice weekly plus sulfasalazine 1000 mg three times daily
A
  1. Answer: A
    According to the ACR and GRAPPA, patients with minimal to no functional limitations from PsA should be treated
    only with NSAIDs or other analgesics. When the symptoms progress to moderate severity and affect the patient’s
    activities of daily living, or when the symptoms do not
    respond to simple analgesics, providers should consider
    adding either a DMARD (e.g., sulfasalazine) or a biologic
    agent (e.g., etanercept). Combination DMARD and biologic
    agent should be reserved for patients with severe disease or
    for those whose condition does not respond to either agent
    alone (Answer A is correct; Answers B-D are incorrect)
How well did you know this?
1
Not at all
2
3
4
5
Perfectly
7
Q
  1. J.O. is a 76-year-old woman with a history of type 2
    diabetes and chronic stable angina (medically managed). She has bilateral knee osteoarthritis (OA) pain
    that has not been sufficiently controlled with physical
    therapy or acetaminophen 1000 mg every 6 hours. She
    cannot perform many activities of daily living because
    she requires a walker, which considerably impairs her
    mobility. Which regimen is best to help alleviate the
    patient’s chronic pain?
    A. Meloxicam 7.5 mg once daily.
    B. Topical diclofenac 1% gel.
    C. Ketorolac 10 mg every 6 hours.
    D. Morphine sulfate extended release 15 mg twice
    daily
A
  1. Answer: B
    For this patient, the next best choice for pain relief is topical
    diclofenac 1% gel. The ACR 2012 guidelines do not recommend the routine use of opiate analgesia for OA pain. In
    addition, given her history of ischemic heart disease, she
    should avoid using meloxicam or oral diclofenac because
    of an FDA report regarding NSAID use and risk of CV
    events. Although diclofenac gel is an NSAID, its topical
    application limits the amount of systemic absorption and
    possibly systemic adverse events (Answer B is correct;
    Answers A, C and D are incorrect). Patients who use topical NSAIDs are at a higher risk of dermatologic reactions
    than those who use systemic NSAIDs.
How well did you know this?
1
Not at all
2
3
4
5
Perfectly
8
Q
  1. T.Q. is a 29-year-old woman without obesity who has
    been treated with hydroxychloroquine for systemic
    lupus erythematosus (SLE) for the past 3 years. Her
    current dose is 400 mg once daily (about 5.4 mg/kg).
    She speaks with her pharmacist, who asks whether she
    has been receiving regular ophthalmologic screenings
    for patients chronically treated with hydroxychloroquine. The patient has never had her eyes checked.
    Which would be the best recommendation for this
    patient’s current and future ophthalmologic screening?
    A. Initial screening now and then every 5 years.
    B. Initial screening now and then annually thereafter.
    C. Initial screening now and then annually starting
    at year 5.
    D. Initial screening now and then every 6 months
    starting at year 5
A
  1. Answer: B
    Even though the patient has used hydroxychloroquine for
    less than 5 years, her current daily dose is greater than
    5 mg/kg/day, placing her in a higher-risk category for
    hydroxychloroquine-related ocular complications. People
    in the major risk category should have a baseline, followed
    by annual, funduscopic examination. (Answer B is correct;
    Answers A, C and D are incorrect)
How well did you know this?
1
Not at all
2
3
4
5
Perfectly
9
Q
  1. R.V. is a 42-year-old woman with a significant history
    of depression and schizophrenia. Her current drug regimen is ziprasidone 40 mg twice daily and selegiline
    transdermal 6 mg/24 hours. Her symptoms are consistent with fibromyalgia syndrome, but she has been
    reluctant to start treatment until now because she was
    afraid it would interfere with her other mental health
    medications. However, the symptoms have worsened
    during the past 6 months, and she now asks to begin
    therapy. Which medication would be the best for R.V.
    to begin taking?
    A. Nortriptyline 25 mg once daily in the evening.
    B. Gabapentin 100 mg twice daily.
    C. Pregabalin 75 mg twice daily.
    D. Duloxetine 60 mg once daily.
A
  1. Answer: C
    The patient would best begin treatment with pregabalin
    75 mg twice daily. Although all the medications listed are
    appropriate for treating fibromyalgia syndrome, several
    issues need to be considered. Nortriptyline and duloxetine would create a significant drug-drug interaction with
    transdermal selegiline because it inhibits both monoamine
    oxidase A and monoamine oxidase B (nonselective), most
    likely resulting in hypertensive crisis and/or serotonin syndrome. The gabapentin dose is too low for the patient and
    would most likely not have a clinically significant change
    in her symptoms. The target dose for gabapentin for fibromyalgia is 1800–2400 mg daily (divided three times).
    (Answer C is correct; Answers A, B and D are incorrect)
How well did you know this?
1
Not at all
2
3
4
5
Perfectly
10
Q
  1. L.L. is a 58-year-old man with chronic tophaceous
    gout and stage 4 chronic kidney disease (CKD). He
    reports only taking over-the-counter (OTC) ibuprofen for gout in the past but states he was told to stop
    because it “hurt his kidneys.” He states that the tophi
    sometimes bother him and that he has two or three
    attacks per year. The patient has 10–12 alcoholic
    drinks a day and regularly consumes a lot of meat proteins. In addition to dietary counseling, which therapy
    is best to decrease tophi and prevent gouty attacks in
    this patient?
    A. No therapy is required until he has two or more
    gouty attacks in a 12-month period.
    B. Administer allopurinol 50 mg once daily, slowly
    titrated over the next few months.
    C. Administer allopurinol 300 mg once daily, slowly
    titrated over the next few months.
    D. Administer colchicine 0.6 mg three times weekly
A
  1. Answer: B
    The patient has CKD, thereby limiting the choice of medications and doses that can be used to prevent recurrent
    gouty attacks. The patient is a candidate for gout prevention
    and treatment of hyperuricemia. The number of attacks per
    year does not factor in when initiating therapy. Colchicine
    does not affect tophi formation. Xanthine oxidase inhibitors (allopurinol first line) are the drug of choice for patients
    with tophi. The ACR guidelines state that in patients with
    stage 4 CKD or worse, allopurinol therapy can be initiated at 50 mg/day, increasing the dose every 2–5 weeks to
    achieve the desired uric acid concentrations; doses greater
    than 300 mg/day are allowed but with appropriate patient
    education and monitoring for toxicity (Answer B is correct;
    Answers A, C and D are incorrect)
How well did you know this?
1
Not at all
2
3
4
5
Perfectly